Mathcenter Forum  

Go Back   Mathcenter Forum > คณิตศาสตร์โอลิมปิก และอุดมศึกษา > ทฤษฎีจำนวน
สมัครสมาชิก คู่มือการใช้ รายชื่อสมาชิก ปฏิทิน ข้อความวันนี้

ตั้งหัวข้อใหม่ Reply
 
เครื่องมือของหัวข้อ ค้นหาในหัวข้อนี้
  #1  
Old 12 ตุลาคม 2011, 20:01
Metamorphosis's Avatar
Metamorphosis Metamorphosis ไม่อยู่ในระบบ
ลมปราณคุ้มครองร่าง
 
วันที่สมัครสมาชิก: 26 กรกฎาคม 2011
ข้อความ: 312
Metamorphosis is on a distinguished road
Default พิสูจน์ให้ดูหน่อยครับ

กำหนด $n>1$ และ $n$ เป็นจำนวนนับ
จงพิสูจน์ว่า $n!$ ไม่สามารถเขียนอยู่ในรูปกำลังสองได้่
__________________
Fighting for Eng.CU
ตอบพร้อมอ้างอิงข้อความนี้
  #2  
Old 12 ตุลาคม 2011, 20:22
กระบี่ทะลวงด่าน's Avatar
กระบี่ทะลวงด่าน กระบี่ทะลวงด่าน ไม่อยู่ในระบบ
กระบี่ไว
 
วันที่สมัครสมาชิก: 07 พฤศจิกายน 2010
ข้อความ: 227
กระบี่ทะลวงด่าน is on a distinguished road
Default

ลองใช้จำนวนเฉพาะที่มากที่สุดที่ < หรือ = n สิครับ
__________________
God does mathematics.
ตอบพร้อมอ้างอิงข้อความนี้
  #3  
Old 12 ตุลาคม 2011, 21:18
Metamorphosis's Avatar
Metamorphosis Metamorphosis ไม่อยู่ในระบบ
ลมปราณคุ้มครองร่าง
 
วันที่สมัครสมาชิก: 26 กรกฎาคม 2011
ข้อความ: 312
Metamorphosis is on a distinguished road
Default

ผมทำถูกไหมครับ

เนื่องจาก $n!>1$ จะได้ว่า มี $p \left|\,\right. n!$ ลงตัว
ให้ $p_n$ แทนจำนวนเฉพาะที่มากที่สุดที่หาร $n!$ ลงตัว

ให้ $n!=1 \cdot 2 \cdot 3 \cdot \cdot \cdot p_n \cdot \cdot n$

ุแบ่งกรณี
ถ้า $p_n = n$ เห็นได้ชัดเจนว่า ไม่เป็นกำลังสองสมบูรณ์
ถ้า $p_n < n$ จะได้ $p_n+1 , p_n+2 , .... , n$ สามารถเขียนอยู่ในรูปจำนวนเต็มบวกคูณกันสองจำนวนคือ a และ b โดย $1<a\leqslant b$ เราก็จะได้ผลตามมาคือ จะมี $p \left|\,\right. a$ และ $p \left|\,\right. b$

เราจะพิสูจน์ว่าไม่มี $p_n+k$ ที่สามารถเขียนอยู่ในรูป $p_n \cdot r$ โดย $r$ เป็นจำนวนเต็มบวก
สมมุติว่าสามารถเขียนได้จริง $p_n + k = p_n \cdot r$ โดย $k < p_n$ (เพราะว่า ถ้า $k\geqslant p_n$ จะได้ว่ามีจำนวนเฉพาะใน $p_n < p < p_n+k$)
$1+\dfrac{k}{p_n} = r$ จะได้ $k = 0$ เท่านั้น ส่งผลให้ $r=1$
สรุปคือ เราไม่สามารถเขียน $n!$ ในอยู่ในรูป $p_n$ มากกว่า 1 จำนวนได้ เมื่อ $n>1$
__________________
Fighting for Eng.CU

12 ตุลาคม 2011 21:24 : ข้อความนี้ถูกแก้ไขแล้ว 2 ครั้ง, ครั้งล่าสุดโดยคุณ Metamorphosis
ตอบพร้อมอ้างอิงข้อความนี้
  #4  
Old 12 ตุลาคม 2011, 22:04
PP_nine's Avatar
PP_nine PP_nine ไม่อยู่ในระบบ
กระบี่ประสานใจ
 
วันที่สมัครสมาชิก: 24 เมษายน 2010
ข้อความ: 607
PP_nine is on a distinguished road
Default

ใช้ Chebychev's Theorem ที่จะมีจำนวนเฉพาะ $p$ ซึ่ง $a<p<2a$ เสมอทุก $a>1$

อันนี้น่าจะง่ายกว่านะครับ
__________________
keep your way.
ตอบพร้อมอ้างอิงข้อความนี้
  #5  
Old 12 ตุลาคม 2011, 22:47
Amankris's Avatar
Amankris Amankris ไม่อยู่ในระบบ
กระบี่ธรรมชาติ
 
วันที่สมัครสมาชิก: 13 มกราคม 2007
ข้อความ: 2,492
Amankris is on a distinguished road
Default

@#4
#3 เค้าก็ใช้นะครับ (ถ้าไม่ใช้คงทำได้ยุ่งยากกว่านี้)
ตอบพร้อมอ้างอิงข้อความนี้
  #6  
Old 12 ตุลาคม 2011, 23:02
PP_nine's Avatar
PP_nine PP_nine ไม่อยู่ในระบบ
กระบี่ประสานใจ
 
วันที่สมัครสมาชิก: 24 เมษายน 2010
ข้อความ: 607
PP_nine is on a distinguished road
Default

เห็นเขียนเยอะเลยนึกว่าไม่ได้ใช้น่ะครับ มึน
__________________
keep your way.
ตอบพร้อมอ้างอิงข้อความนี้
  #7  
Old 13 ตุลาคม 2011, 17:37
Metamorphosis's Avatar
Metamorphosis Metamorphosis ไม่อยู่ในระบบ
ลมปราณคุ้มครองร่าง
 
วันที่สมัครสมาชิก: 26 กรกฎาคม 2011
ข้อความ: 312
Metamorphosis is on a distinguished road
Default

ช่วยพิสูจน์อันนี้ให้หน่อยครับ


1. จงแสดงว่าถ้า $n \in \mathbb{N} $ ซึ่ง $n>2$แล้วจะมีจำนวนเฉพาะ $p$ ที่สอดคล้องกับ n$<p<n!$ (ผมพิสูจน์ว่า n!>2n แทนได้ไหมครับ แล้วอ้าง chebychev)

2. ให้ $p_n$ แทนจำนวนเฉพาะตัวที่ $n$ แล้วจงแสดงว่า $p_n > 2n-1$ สำหรับ $n \geqslant 5$

3. ให้ $p_1,p_2,p_3,....p_n$ เป็นลำดับของจำนวนเฉพาะ จงตรวจสอบว่า
$P^* = p_1p_2p_3 \cdot \cdot \cdot \cdot p_n +1$ เป็นกำลังสองสมบูรณ์หรือไม่ (ผมใช้ mod 4 อะครับ ได้ปะครับ)
__________________
Fighting for Eng.CU

13 ตุลาคม 2011 18:50 : ข้อความนี้ถูกแก้ไขแล้ว 2 ครั้ง, ครั้งล่าสุดโดยคุณ Metamorphosis
ตอบพร้อมอ้างอิงข้อความนี้
  #8  
Old 14 ตุลาคม 2011, 09:28
nooonuii nooonuii ไม่อยู่ในระบบ
ผู้พิทักษ์กฎทั่วไป
 
วันที่สมัครสมาชิก: 25 พฤษภาคม 2001
ข้อความ: 6,408
nooonuii is on a distinguished road
Default

$1,3$ ทำได้ครับ

2. ใช้ induction ดูรึยัง
__________________
site:mathcenter.net คำค้น
ตอบพร้อมอ้างอิงข้อความนี้
  #9  
Old 14 ตุลาคม 2011, 11:45
Metamorphosis's Avatar
Metamorphosis Metamorphosis ไม่อยู่ในระบบ
ลมปราณคุ้มครองร่าง
 
วันที่สมัครสมาชิก: 26 กรกฎาคม 2011
ข้อความ: 312
Metamorphosis is on a distinguished road
Default

ขอบคุณครับ

ถ้าจะพิสูจน์ 3 แบบไม่ใช้ modอะครับ ช่วยแสดงให้หน่อยครับ
__________________
Fighting for Eng.CU

14 ตุลาคม 2011 12:11 : ข้อความนี้ถูกแก้ไขแล้ว 1 ครั้ง, ครั้งล่าสุดโดยคุณ Metamorphosis
ตอบพร้อมอ้างอิงข้อความนี้
  #10  
Old 14 ตุลาคม 2011, 13:29
nooonuii nooonuii ไม่อยู่ในระบบ
ผู้พิทักษ์กฎทั่วไป
 
วันที่สมัครสมาชิก: 25 พฤษภาคม 2001
ข้อความ: 6,408
nooonuii is on a distinguished road
Default

อ้างอิง:
ข้อความเดิมเขียนโดยคุณ Metamorphosis View Post
3. ให้ $p_1,p_2,p_3,....p_n$ เป็นลำดับของจำนวนเฉพาะ จงตรวจสอบว่า
$P^* = p_1p_2p_3 \cdot \cdot \cdot \cdot p_n +1$ เป็นกำลังสองสมบูรณ์หรือไม่ (ผมใช้ mod 4 อะครับ ได้ปะครับ)
กรณี $n=1$ เห็นได้ชัด สมมติ $n\geq 2$

สังเกตว่า $P^*$ เป็นเลขคี่

สมมติ $P^*=(2m+1)^2$ จะได้ว่า

$p_1p_2p_3\cdots p_n=4(m^2+m)$

$p_2p_3\cdots p_n=2(m^2+m)$

ซึ่งขัดแย้ง
__________________
site:mathcenter.net คำค้น

14 ตุลาคม 2011 13:33 : ข้อความนี้ถูกแก้ไขแล้ว 1 ครั้ง, ครั้งล่าสุดโดยคุณ nooonuii
ตอบพร้อมอ้างอิงข้อความนี้
  #11  
Old 14 ตุลาคม 2011, 21:58
Metamorphosis's Avatar
Metamorphosis Metamorphosis ไม่อยู่ในระบบ
ลมปราณคุ้มครองร่าง
 
วันที่สมัครสมาชิก: 26 กรกฎาคม 2011
ข้อความ: 312
Metamorphosis is on a distinguished road
Default

1. Proof that there exist infinitely many positive integers n such that $n\left|\,\right. 2^n+2$

2. สำหรับ $n>1$ จงแสดงว่า ทุกจำนวนเฉพาะที่หาร $n!+1$ ลงตัวจะเป็นจำนวนเต็มคี่ที่มากกว่า $n$

ช่วยดูข้อ 2 หน่อยครับ
สังเกตว่า $n>1, n!+1$ จะเป็นจำนวนเต็มบวกคี่
ถ้า $n!+1$ เป็นจำนวนเฉพาะ จะได้ว่า มีจำนวนเฉพาะที่มากกว่า $n$ ซึ่งหาร $n!+1$ ลงตัว
ถ้า $n!+1$ เป็นจำนวนประกอบ ให้ $n!+1 = p_1 \cdot p_2 \cdot p_3 \cdot \cdot \cdot \cdot p_k+1$
ให้ $p_i$ เป็นจำนวนเฉพาะซึ่งหาร $n!+1$ ลงตัว ถ้า $1 \leqslant i \leqslant k$ จะได้ว่า $p_i \left|\,\right. 1$ ซึ่งเป็นไปไม่ได้
ถ้า $p_i < n$ จะได้ว่า $p_i\left|\,\right. n!$ ซึ่งเราได้พิสูจน์มาก่อนหน้านี้แล้วว่า $p_i$ ไม่ได้อยู่ในลำดับ $n!$
แสดงว่า $p_i > n$
__________________
Fighting for Eng.CU

14 ตุลาคม 2011 22:32 : ข้อความนี้ถูกแก้ไขแล้ว 2 ครั้ง, ครั้งล่าสุดโดยคุณ Metamorphosis
ตอบพร้อมอ้างอิงข้อความนี้
  #12  
Old 15 ตุลาคม 2011, 00:13
nooonuii nooonuii ไม่อยู่ในระบบ
ผู้พิทักษ์กฎทั่วไป
 
วันที่สมัครสมาชิก: 25 พฤษภาคม 2001
ข้อความ: 6,408
nooonuii is on a distinguished road
Default

อ้างอิง:
ข้อความเดิมเขียนโดยคุณ Metamorphosis View Post

2. สำหรับ $n>1$ จงแสดงว่า ทุกจำนวนเฉพาะที่หาร $n!+1$ ลงตัวจะเป็นจำนวนเต็มคี่ที่มากกว่า $n$

ช่วยดูข้อ 2 หน่อยครับ
สังเกตว่า $n>1, n!+1$ จะเป็นจำนวนเต็มบวกคี่
ถ้า $n!+1$ เป็นจำนวนเฉพาะ จะได้ว่า มีจำนวนเฉพาะที่มากกว่า $n$ ซึ่งหาร $n!+1$ ลงตัว
ถ้า $n!+1$ เป็นจำนวนประกอบ ให้ $n!+1 = p_1 \cdot p_2 \cdot p_3 \cdot \cdot \cdot \cdot p_k+1$
ให้ $p_i$ เป็นจำนวนเฉพาะซึ่งหาร $n!+1$ ลงตัว ถ้า $1 \leqslant i \leqslant k$ จะได้ว่า $p_i \left|\,\right. 1$ ซึ่งเป็นไปไม่ได้
ถ้า $p_i < n$ จะได้ว่า $p_i\left|\,\right. n!$ ซึ่งเราได้พิสูจน์มาก่อนหน้านี้แล้วว่า $p_i$ ไม่ได้อยู่ในลำดับ $n!$
แสดงว่า $p_i > n$
สีแดงไม่จริงครับ ถ้า $p_i$ หมายถึงจำนวนเฉพาะตัวที่ $i$ และทำอะไรยุ่งยากกว่าที่โจทย์กำหนด

ถ้าอ่านโจทย์ให้ดีจะต้องเริ่มแบบนี้

สมมติ $p$ เป็นจำนวนเฉพาะ และ $p|(n!+1)$

ถ้า $p\leq n$ จะเกิดอะไรขึ้นครับ ?

อีกสองบรรทัดจบ
__________________
site:mathcenter.net คำค้น
ตอบพร้อมอ้างอิงข้อความนี้
  #13  
Old 15 ตุลาคม 2011, 09:57
Metamorphosis's Avatar
Metamorphosis Metamorphosis ไม่อยู่ในระบบ
ลมปราณคุ้มครองร่าง
 
วันที่สมัครสมาชิก: 26 กรกฎาคม 2011
ข้อความ: 312
Metamorphosis is on a distinguished road
Default

อ้างอิง:
ข้อความเดิมเขียนโดยคุณ nooonuii View Post
สีแดงไม่จริงครับ ถ้า $p_i$ หมายถึงจำนวนเฉพาะตัวที่ $i$ และทำอะไรยุ่งยากกว่าที่โจทย์กำหนด

ถ้าอ่านโจทย์ให้ดีจะต้องเริ่มแบบนี้

สมมติ $p$ เป็นจำนวนเฉพาะ และ $p|(n!+1)$

ถ้า $p\leq n$ จะเกิดอะไรขึ้นครับ ?

อีกสองบรรทัดจบ
$p_i$ ไม่ใช่จำนวนเฉพาะตัวที่ $i$ ครับ แล้วทำไมไม่จริงหรอครับ

สำหรับคำแนะนำขอบคุณมากครับ
__________________
Fighting for Eng.CU
ตอบพร้อมอ้างอิงข้อความนี้
  #14  
Old 15 ตุลาคม 2011, 11:52
nooonuii nooonuii ไม่อยู่ในระบบ
ผู้พิทักษ์กฎทั่วไป
 
วันที่สมัครสมาชิก: 25 พฤษภาคม 2001
ข้อความ: 6,408
nooonuii is on a distinguished road
Default

อ้างอิง:
ข้อความเดิมเขียนโดยคุณ Metamorphosis View Post
$p_i$ ไม่ใช่จำนวนเฉพาะตัวที่ $i$ ครับ แล้วทำไมไม่จริงหรอครับ

สำหรับคำแนะนำขอบคุณมากครับ
แล้ว $p_i$ คืออะไรเหรอครับ

ถ้าเป็นจำนวนเฉพาะก็คงไม่จริงอยู่ดีเพราะ $n!$ มีตัวประกอบเยอะกว่านั้นมากครับ

เช่น $4!=2^3\cdot 3$
__________________
site:mathcenter.net คำค้น

15 ตุลาคม 2011 11:54 : ข้อความนี้ถูกแก้ไขแล้ว 2 ครั้ง, ครั้งล่าสุดโดยคุณ nooonuii
ตอบพร้อมอ้างอิงข้อความนี้
  #15  
Old 15 ตุลาคม 2011, 12:11
Metamorphosis's Avatar
Metamorphosis Metamorphosis ไม่อยู่ในระบบ
ลมปราณคุ้มครองร่าง
 
วันที่สมัครสมาชิก: 26 กรกฎาคม 2011
ข้อความ: 312
Metamorphosis is on a distinguished road
Default

อ้างอิง:
ข้อความเดิมเขียนโดยคุณ nooonuii View Post
แล้ว $p_i$ คืออะไรเหรอครับ

ถ้าเป็นจำนวนเฉพาะก็คงไม่จริงอยู่ดีเพราะ $n!$ มีตัวประกอบเยอะกว่านั้นมากครับ

เช่น $4!=2^3\cdot 3$
ก็คือ เราสมมุติให้ $p_i$ อยู่ใน $p_1,p_2,..p_k$ ซักตัวนึง

เช่น $4! = 2*2*2*3$

คือ $p_1 =2, p_2 =2 ,p_3 =2, p_4=3$ เราให้ $p_i$ หาร $4!+1$ ลงตัว แสดงว่า $p_i$ ต้องหาร $p_1p_2p_3p_4$ และ $1$ ลงตัวด้วยเช่นกันซึ่งเป็นไปไม่ได้อะครับ ผมเข้าใจอะไรผิดไปไหม
ตอบพร้อมอ้างอิงข้อความนี้
ตั้งหัวข้อใหม่ Reply



กฎการส่งข้อความ
คุณ ไม่สามารถ ตั้งหัวข้อใหม่ได้
คุณ ไม่สามารถ ตอบหัวข้อได้
คุณ ไม่สามารถ แนบไฟล์และเอกสารได้
คุณ ไม่สามารถ แก้ไขข้อความของคุณเองได้

vB code is On
Smilies are On
[IMG] code is On
HTML code is Off
ทางลัดสู่ห้อง


เวลาที่แสดงทั้งหมด เป็นเวลาที่ประเทศไทย (GMT +7) ขณะนี้เป็นเวลา 04:00


Powered by vBulletin® Copyright ©2000 - 2024, Jelsoft Enterprises Ltd.
Modified by Jetsada Karnpracha